Question and Answers Forum

All Questions      Topic List

Mechanics Questions

Previous in All Question      Next in All Question      

Previous in Mechanics      Next in Mechanics      

Question Number 144496 by ajfour last updated on 25/Jun/21

Commented by ajfour last updated on 25/Jun/21

A rod is to be released   horizontally at height y   such that it hits ground  in vertical orientation.  Find y in terms of a,b,h.  Rod suffers an elastic  collision with the corner  of table.

$${A}\:{rod}\:{is}\:{to}\:{be}\:{released}\: \\ $$$${horizontally}\:{at}\:{height}\:{y}\: \\ $$$${such}\:{that}\:{it}\:{hits}\:{ground} \\ $$$${in}\:{vertical}\:{orientation}. \\ $$$${Find}\:{y}\:{in}\:{terms}\:{of}\:{a},{b},{h}. \\ $$$${Rod}\:{suffers}\:{an}\:{elastic} \\ $$$${collision}\:{with}\:{the}\:{corner} \\ $$$${of}\:{table}. \\ $$

Answered by mr W last updated on 27/Jun/21

Commented by mr W last updated on 27/Jun/21

u=(√(2gy))  (m/2)(u^2 −v^2 )=(1/2)×((mb^2 )/(12))×ω^2   ⇒12(u^2 −v^2 )=b^2 ω^2    ...(i)  mu−P=mv  P((b/2)−a)=((mb^2 )/(12))ω  m(u−v)((b/2)−a)=((mb^2 )/(12))ω  ⇒ 6(u−v)(b−2a)=b^2 ω   ...(ii)  (i)/(ii):  ωb=((2(u+v))/(1−((2a)/b)))  put into (ii):  6(u−v)(b−2a)=b((2(u+v))/(1−((2a)/b)))  3(1−((2a)/b))^2 (u−v)=u+v  ⇒v=((3(1−((2a)/b))^2 −1)/(3(1−((2a)/b))^2 +1))u=βu  v+u=((6(1−((2a)/b))^2 )/(3(1−((2a)/b))^2 +1))u  ⇒bω=((12(1−((2a)/b)))/(3(1−((2a)/b))^2 +1))u=αu  vt+(1/2)gt^2 =h−(b/2)  2vt+gt^2 =2h−b  t=((−v+(√(v^2 +g(2h−b))))/g)  ωt=(π/2)  2ω[(√(v^2 +g(2h−b)))−v]=πg   ...(iii)  2α[u(√(β^2 u^2 +g(2h−b)))−βu^2 ]=πgb  u(√(β^2 u^2 +g(2h−b)))=βu^2 +((πgb)/(2α))  (2h−b−((βπb)/α))u^2 =((π^2 gb^2 )/(4α^2 ))  (2h−b−((βπb)/α))2gy=((π^2 gb^2 )/(4α^2 ))  ⇒y=((π^2 b)/(8α^2 (((2h)/b)−1−((βπ)/α))))  with   α=((12(1−((2a)/b)))/(3(1−((2a)/b))^2 +1))  β=((3(1−((2a)/b))^2 −1)/(3(1−((2a)/b))^2 +1))

$${u}=\sqrt{\mathrm{2}{gy}} \\ $$$$\frac{{m}}{\mathrm{2}}\left({u}^{\mathrm{2}} −{v}^{\mathrm{2}} \right)=\frac{\mathrm{1}}{\mathrm{2}}×\frac{{mb}^{\mathrm{2}} }{\mathrm{12}}×\omega^{\mathrm{2}} \\ $$$$\Rightarrow\mathrm{12}\left({u}^{\mathrm{2}} −{v}^{\mathrm{2}} \right)={b}^{\mathrm{2}} \omega^{\mathrm{2}} \:\:\:...\left({i}\right) \\ $$$${mu}−{P}={mv} \\ $$$${P}\left(\frac{{b}}{\mathrm{2}}−{a}\right)=\frac{{mb}^{\mathrm{2}} }{\mathrm{12}}\omega \\ $$$${m}\left({u}−{v}\right)\left(\frac{{b}}{\mathrm{2}}−{a}\right)=\frac{{mb}^{\mathrm{2}} }{\mathrm{12}}\omega \\ $$$$\Rightarrow\:\mathrm{6}\left({u}−{v}\right)\left({b}−\mathrm{2}{a}\right)={b}^{\mathrm{2}} \omega\:\:\:...\left({ii}\right) \\ $$$$\left({i}\right)/\left({ii}\right): \\ $$$$\omega{b}=\frac{\mathrm{2}\left({u}+{v}\right)}{\mathrm{1}−\frac{\mathrm{2}{a}}{{b}}} \\ $$$${put}\:{into}\:\left({ii}\right): \\ $$$$\mathrm{6}\left({u}−{v}\right)\left({b}−\mathrm{2}{a}\right)={b}\frac{\mathrm{2}\left({u}+{v}\right)}{\mathrm{1}−\frac{\mathrm{2}{a}}{{b}}} \\ $$$$\mathrm{3}\left(\mathrm{1}−\frac{\mathrm{2}{a}}{{b}}\right)^{\mathrm{2}} \left({u}−{v}\right)={u}+{v} \\ $$$$\Rightarrow{v}=\frac{\mathrm{3}\left(\mathrm{1}−\frac{\mathrm{2}{a}}{{b}}\right)^{\mathrm{2}} −\mathrm{1}}{\mathrm{3}\left(\mathrm{1}−\frac{\mathrm{2}{a}}{{b}}\right)^{\mathrm{2}} +\mathrm{1}}{u}=\beta{u} \\ $$$${v}+{u}=\frac{\mathrm{6}\left(\mathrm{1}−\frac{\mathrm{2}{a}}{{b}}\right)^{\mathrm{2}} }{\mathrm{3}\left(\mathrm{1}−\frac{\mathrm{2}{a}}{{b}}\right)^{\mathrm{2}} +\mathrm{1}}{u} \\ $$$$\Rightarrow{b}\omega=\frac{\mathrm{12}\left(\mathrm{1}−\frac{\mathrm{2}{a}}{{b}}\right)}{\mathrm{3}\left(\mathrm{1}−\frac{\mathrm{2}{a}}{{b}}\right)^{\mathrm{2}} +\mathrm{1}}{u}=\alpha{u} \\ $$$${vt}+\frac{\mathrm{1}}{\mathrm{2}}{gt}^{\mathrm{2}} ={h}−\frac{{b}}{\mathrm{2}} \\ $$$$\mathrm{2}{vt}+{gt}^{\mathrm{2}} =\mathrm{2}{h}−{b} \\ $$$${t}=\frac{−{v}+\sqrt{{v}^{\mathrm{2}} +{g}\left(\mathrm{2}{h}−{b}\right)}}{{g}} \\ $$$$\omega{t}=\frac{\pi}{\mathrm{2}} \\ $$$$\mathrm{2}\omega\left[\sqrt{{v}^{\mathrm{2}} +{g}\left(\mathrm{2}{h}−{b}\right)}−{v}\right]=\pi{g}\:\:\:...\left({iii}\right) \\ $$$$\mathrm{2}\alpha\left[{u}\sqrt{\beta^{\mathrm{2}} {u}^{\mathrm{2}} +{g}\left(\mathrm{2}{h}−{b}\right)}−\beta{u}^{\mathrm{2}} \right]=\pi{gb} \\ $$$${u}\sqrt{\beta^{\mathrm{2}} {u}^{\mathrm{2}} +{g}\left(\mathrm{2}{h}−{b}\right)}=\beta{u}^{\mathrm{2}} +\frac{\pi{gb}}{\mathrm{2}\alpha} \\ $$$$\left(\mathrm{2}{h}−{b}−\frac{\beta\pi{b}}{\alpha}\right){u}^{\mathrm{2}} =\frac{\pi^{\mathrm{2}} {gb}^{\mathrm{2}} }{\mathrm{4}\alpha^{\mathrm{2}} } \\ $$$$\left(\mathrm{2}{h}−{b}−\frac{\beta\pi{b}}{\alpha}\right)\mathrm{2}{gy}=\frac{\pi^{\mathrm{2}} {gb}^{\mathrm{2}} }{\mathrm{4}\alpha^{\mathrm{2}} } \\ $$$$\Rightarrow{y}=\frac{\pi^{\mathrm{2}} {b}}{\mathrm{8}\alpha^{\mathrm{2}} \left(\frac{\mathrm{2}{h}}{{b}}−\mathrm{1}−\frac{\beta\pi}{\alpha}\right)} \\ $$$${with}\: \\ $$$$\alpha=\frac{\mathrm{12}\left(\mathrm{1}−\frac{\mathrm{2}{a}}{{b}}\right)}{\mathrm{3}\left(\mathrm{1}−\frac{\mathrm{2}{a}}{{b}}\right)^{\mathrm{2}} +\mathrm{1}} \\ $$$$\beta=\frac{\mathrm{3}\left(\mathrm{1}−\frac{\mathrm{2}{a}}{{b}}\right)^{\mathrm{2}} −\mathrm{1}}{\mathrm{3}\left(\mathrm{1}−\frac{\mathrm{2}{a}}{{b}}\right)^{\mathrm{2}} +\mathrm{1}} \\ $$

Commented by ajfour last updated on 27/Jun/21

Thanks sir, our approach are the same, but you could bring it to completion; excellent soln.

Answered by ajfour last updated on 27/Jun/21

Let impulse received at   corner be J.  u=(√(2gy))  mv=mu−J  −h=vt−((gt^2 )/2)  gt^2 −2vt−2h=0  t=(v/g)+(√((v^2 /g^2 )+((2h)/g)))  ωt=(π/2)  I_0 ω=J((b/2)−a)  ⇒ I_0 ((π/(2t)))=m(u−v)((b/2)−a)  I_0 ((π/2))=((mv(u−v))/g)((b/2)−a)(1+(√(1+((2gh)/v^2 ))))  &   mg(y+h−(b/2))=(1/2)mv^2      +(1/2)I_0 ω^2   and as  y=(u^2 /(2g))  ⇒ mg((u^2 /(2g))+h−(b/2))=(1/2)mv^2     +(1/(2I_0 )){m(u−v)((b/2)−a)}^2  ..(I)  I_0 ((π/2))=  ((mv(u−v))/g)((b/2)−a)(1+(√(1+((2gh)/v^2 ))))     ...(II)  we solve for u and v from  eqs. (I)& (II)  and then    y=(u^2 /(2g))

$${Let}\:{impulse}\:{received}\:{at}\: \\ $$$${corner}\:{be}\:{J}. \\ $$$${u}=\sqrt{\mathrm{2}{gy}} \\ $$$${mv}={mu}−{J} \\ $$$$−{h}={vt}−\frac{{gt}^{\mathrm{2}} }{\mathrm{2}} \\ $$$${gt}^{\mathrm{2}} −\mathrm{2}{vt}−\mathrm{2}{h}=\mathrm{0} \\ $$$${t}=\frac{{v}}{{g}}+\sqrt{\frac{{v}^{\mathrm{2}} }{{g}^{\mathrm{2}} }+\frac{\mathrm{2}{h}}{{g}}} \\ $$$$\omega{t}=\frac{\pi}{\mathrm{2}} \\ $$$${I}_{\mathrm{0}} \omega={J}\left(\frac{{b}}{\mathrm{2}}−{a}\right) \\ $$$$\Rightarrow\:{I}_{\mathrm{0}} \left(\frac{\pi}{\mathrm{2}{t}}\right)={m}\left({u}−{v}\right)\left(\frac{{b}}{\mathrm{2}}−{a}\right) \\ $$$${I}_{\mathrm{0}} \left(\frac{\pi}{\mathrm{2}}\right)=\frac{{mv}\left({u}−{v}\right)}{{g}}\left(\frac{{b}}{\mathrm{2}}−{a}\right)\left(\mathrm{1}+\sqrt{\mathrm{1}+\frac{\mathrm{2}{gh}}{{v}^{\mathrm{2}} }}\right) \\ $$$$\&\:\:\:{mg}\left({y}+{h}−\frac{{b}}{\mathrm{2}}\right)=\frac{\mathrm{1}}{\mathrm{2}}{mv}^{\mathrm{2}} \\ $$$$\:\:\:+\frac{\mathrm{1}}{\mathrm{2}}{I}_{\mathrm{0}} \omega^{\mathrm{2}} \\ $$$${and}\:{as}\:\:{y}=\frac{{u}^{\mathrm{2}} }{\mathrm{2}{g}} \\ $$$$\Rightarrow\:{mg}\left(\frac{{u}^{\mathrm{2}} }{\mathrm{2}{g}}+{h}−\frac{{b}}{\mathrm{2}}\right)=\frac{\mathrm{1}}{\mathrm{2}}{mv}^{\mathrm{2}} \\ $$$$\:\:+\frac{\mathrm{1}}{\mathrm{2}{I}_{\mathrm{0}} }\left\{{m}\left({u}−{v}\right)\left(\frac{{b}}{\mathrm{2}}−{a}\right)\right\}^{\mathrm{2}} \:..\left({I}\right) \\ $$$${I}_{\mathrm{0}} \left(\frac{\pi}{\mathrm{2}}\right)= \\ $$$$\frac{{mv}\left({u}−{v}\right)}{{g}}\left(\frac{{b}}{\mathrm{2}}−{a}\right)\left(\mathrm{1}+\sqrt{\mathrm{1}+\frac{\mathrm{2}{gh}}{{v}^{\mathrm{2}} }}\right) \\ $$$$\:\:\:...\left({II}\right) \\ $$$${we}\:{solve}\:{for}\:{u}\:{and}\:{v}\:{from} \\ $$$${eqs}.\:\left({I}\right)\&\:\left({II}\right)\:\:{and}\:{then} \\ $$$$\:\:{y}=\frac{{u}^{\mathrm{2}} }{\mathrm{2}{g}} \\ $$

Terms of Service

Privacy Policy

Contact: info@tinkutara.com